The passage provides evidence to suggest that the author would be most likely to assent to which one of the following...

mg123 on November 7, 2019

Why is D incorrect?

Why is D incorrect?

Reply
Create a free account to read and take part in forum discussions.

Already have an account? log in

Irina on November 7, 2019

@mg123,

The issue with (D) is that it is too broad of a statement. The passage cites one case where the ruling of a provincial court is excessively conservative. and argues that for this specific case, the aboriginal group is unlikely to be successful unless they go to the Supreme Court There is no evidence to suggest that the author would support taking away the authority to decide any cases involving questions of aboriginal rights from all provincial courts - this would essentially amount to having the Supreme Court deciding even the most trivial cases involving aboriginal rights, which is an absurd result.